LSAT and Law School Admissions Forum

Get expert LSAT preparation and law school admissions advice from PowerScore Test Preparation.

 Administrator
PowerScore Staff
  • PowerScore Staff
  • Posts: 8916
  • Joined: Feb 02, 2011
|
#40799
Complete Question Explanation
(The complete setup for this game can be found here: lsat/viewtopic.php?t=4676)

The correct answer choice is (B)

This List question is easily answered by applying the rules sequentially to the answer choices.

Answer choice (A) is incorrect because it violates the third and fourth rules.

Answer choice (B) is the correct answer.

Answer choice (C) is incorrect because it violates the first rule.

Answer choice (D) is incorrect because it violates the fourth rule.

Answer choice (E) is incorrect because it violates the second rule.

Note that, as is often the case, each of the incorrect answers can be eliminated by a different rule.

Get the most out of your LSAT Prep Plus subscription.

Analyze and track your performance with our Testing and Analytics Package.